[email protected] Real Analysis Analysis_Folland_Ch6.pdfApr 30, 2018  · 2. Prove Theorem 6.8. a. If...

24
myweb.ttu.edu/bban [email protected] Real Analysis Byeong Ho Ban Mathematics and Statistics Texas Tech University Chapter 6. L p spaces(Last update : April 30, 2018) 1. When does equality hold in Minkowski’s inequality? (The answer is different for p =1 and for 1 <p< . What about p = ?) Proof. (April 7th 2018) (1)When p =1 kf + gk 1 = kf k 1 + kgk 1 ⇐⇒ Z |f + g| = Z |f | + Z |g| ⇐⇒ Z (||f + g|- (|f | + |g|)|) ⇐⇒ |f + g| = |f | + |g| a.e. ⇐⇒ ∀ almost every x, α 0 such that f (x)= αg(x) (2) When 1 <p< Since, when f =0 a.e. or g =0 a.e., it is obvious, let’s assume f 6= 0 and g 6=0 a.e. Observe the origin of the inequality below. Z |f + g| p Z (|f | + |g|) |f + g| p-1 ≤kf k p |f + g| p-1 p p-1 + kgk p |f + g| p-1 p p-1 As for the first inequality, it is equality iff a.e.x, α x 0 such that α x f (x)= g(x) As for the second inequality, assuming α x > 0(When α x = 0 it is clear.) it is equality iff |f | p kf k p p = |f + g| p p-1 kf + gk p p-1 p p-1 = |f | p p-1 kf k p p-1 p p-1 and |g| p kgk p p = |f + g| p p-1 kf + gk p p-1 p p-1 = |f | p p-1 kf k p p-1 p p-1 a.e. Thus, kgk p kf k p = α x or α x =0 ! a.e. x 1

Transcript of [email protected] Real Analysis Analysis_Folland_Ch6.pdfApr 30, 2018  · 2. Prove Theorem 6.8. a. If...

  • myweb.ttu.edu/bban

    [email protected]

    Real Analysis

    Byeong Ho BanMathematics and Statistics

    Texas Tech University

    Chapter 6. Lp spaces(Last update : April 30, 2018)

    1. When does equality hold in Minkowski’s inequality? (The answer is different for p = 1 and for 1 < p <∞. What about p =∞?)

    Proof. (April 7th 2018)

    (1)When p = 1

    ‖f + g‖1 = ‖f‖1 + ‖g‖1 ⇐⇒∫|f + g| =

    ∫|f |+

    ∫|g|

    ⇐⇒∫

    (||f + g| − (|f |+ |g|)|)

    ⇐⇒ |f + g| = |f |+ |g| a.e.⇐⇒ ∀ almost every x,∃α ≥ 0 such that f(x) = αg(x)

    (2) When 1 < p 0(When αx = 0 it is clear.) it is equality iff

    |f |p

    ‖f‖pp=|f + g|

    pp−1

    ‖f + g‖pp−1pp−1

    =|f |

    pp−1

    ‖f‖pp−1pp−1

    and|g|p

    ‖g‖pp=|f + g|

    pp−1

    ‖f + g‖pp−1pp−1

    =|f |

    pp−1

    ‖f‖pp−1pp−1

    a.e.

    Thus, (‖g‖p‖f‖p

    = αx or αx = 0

    )a.e. x

    1

  • 2

    (3) When p =∞

    ‖f + g‖∞ = ‖f‖∞ + ‖g‖∞ ⇐⇒ ‖f + g‖∞ ≥ |f |+ |g| a.e.⇐⇒ |f + g| ≥ |f |+ |g| a.e.⇐⇒ f = αxg a.e.

    where αx ≥ 0. �

    2. Prove Theorem 6.8.

    a. If f and g are measurable functions on X, then ‖fg‖1 ≤ ‖f‖1 ‖g‖∞. If f ∈ L1 and g ∈ L∞,‖fg‖1 = ‖f‖1 ‖g‖∞ iff |g(x)| = ‖g‖∞ a.e. on the set where f(x) 6= 0.

    b. ‖·‖∞ is a norm on L∞

    c. ‖fn − f‖∞ → 0 iff there exists E ∈M such that µ(Ec) = 0 and fn → f uniformly on E.

    d. L∞ is a Banach space.

    e. The simple functions are dense in L∞.

    Proof. (April 7th 2018)

    a.Observe that

    |fg| = |f ||g| ≤ |f | ‖g‖∞ =⇒ ‖fg‖1 =∫|fg| ≤ ‖g‖∞

    ∫|f | = ‖f‖1 ‖g‖∞

    Suppose that f ∈ L1 and g ∈ L∞.Observe that

    ‖fg‖1 = ‖f‖1 ‖g‖∞ ⇐⇒∫|fg| = ‖g‖∞

    ∫|f |

    ⇐⇒∫|f |(||g| − ‖g‖∞ |) = 0

    ⇐⇒ |g(x)| = ‖g‖∞ a.e. on E = {x : f(x) 6= 0}

    b.(1)

    ‖f‖∞ = 0 ⇐⇒ |f(x)| ≤ 0 a.e.⇐⇒ f = 0 a.e.

  • 3

    (2) if λ = 0, it is obvious, so let’s assume λ 6= 0.

    |λf | = |λ||f | ≤ |λ| ‖f‖∞ =⇒ ‖λf‖∞ ≤ |λ| ‖f‖∞|λ||f | = |λf | ≤ ‖λf‖∞ =⇒ |λ| ‖f‖∞ ≤ ‖λf‖∞

    (3)

    |f + g| ≤ |f |+ |g| ≤ ‖f‖∞ + ‖g‖∞ =⇒ ‖f + g‖∞ ≤ ‖f‖∞ + ‖g‖∞

    c. Let � > 0 be given.Suppose that ‖fn − f‖∞ → 0.Let

    En = {x : |fn(x)− f(x)| > ‖fn − f‖∞}c E =

    ∞⋂n=1

    En

    Note that µ(Ecn) = 0 ∀n. so µ(Ec) = 0.And observe that, there is N ∈ N such that

    |fn(x)− f | ≤ ‖fn − f‖∞ < � ∀n ≥ N∀x ∈ E

    Therefore, fn → f uniformly on E.

    Conversely, suppose that fn → f on E ⊂ X with µ(Ec) = 0. Then, there is N ∈ N such that

    |fn(x)− f(x)| < � ∀n ≥ N∀x ∈ E

    Then

    ‖fn − f‖∞ < � ∀n ≥ N

    d.Let {fn} ⊂ L∞ such that

    ∞∑n=1

    ‖fn‖∞ ‖fn‖∞} E =∞⋂n=1

    En

    Note that µ(Ecn) = 0, so µ(Ec) = 0 and that

    ∞∑n=1

    |fn(x)| ≤∞∑n=1

    ‖fn‖∞

  • 4

    Now, observe that, for any m ∈ N∣∣∣∣∣∞∑

    n=m+1

    fn(x)

    ∣∣∣∣∣ ≤∞∑

    n=m+1

    |fn(x)| ≤∞∑

    n=m+1

    ‖fn‖∞

    Since ∥∥∥∥∥∞∑

    n=m+1

    fn(x)

    ∥∥∥∥∥∞

    ≤∞∑

    n=m+1

    ‖fn‖∞ → 0 as m→∞

    the series converges in L∞, so L∞ is a Banach space.

    e.Let f ∈ L∞, then there exists a sequence of simple functions {fn} such that

    fn → f a.e, |fn| ≤ |f |Thus, there exists E such that fn → f uniformly on E and µ(Ec) = 0. Thus, by c.,

    ‖fn − f‖∞ → 0 as n→∞therefore, the space of simple function is dense in L∞

    3. If 1 ≤ p ≤ r ≤ ∞, Lp ∩ Lr is a Banach space with norm‖f‖ = ‖f‖p + ‖f‖r

    and if p < q < r, the inclusion map Lp ∩ Lr → Lq is continuous.

    Proof. (April 8th 2018)

    First of all, note that Lp ∩ Lr is a vector space. And observe the following argument.‖f‖ = 0 ⇐⇒ ‖f‖p + ‖f‖r = 0

    ⇐⇒ ‖f‖p = 0 ∧ ‖f‖r = 0⇐⇒ f = 0

    ‖λf‖ = ‖λf‖p + ‖λf‖r = |λ|(‖f‖p + ‖f‖r

    )= |λ| ‖f‖

    ‖f + g‖ = ‖f + g‖p + ‖f + g‖r ≤ ‖f‖p + ‖g‖p + ‖f‖r + ‖g‖r = ‖f‖+ ‖g‖

    Thus, Lp ∩ Lr is a normed vector space with the norm ‖·‖.Let {fn} ⊂ Lp ∩ Lr be a sequence such that

    ∞∑n=1

    ‖fn‖

  • 5

    Thus, ∥∥∥∥∥∞∑

    n=m+1

    fn

    ∥∥∥∥∥p

    → 0

    ∥∥∥∥∥∞∑

    n=m+1

    fn

    ∥∥∥∥∥r

    → 0

    as m→∞.Therefore, ∥∥∥∥∥

    ∞∑n=m+1

    fn

    ∥∥∥∥∥ =∥∥∥∥∥

    ∞∑n=m+1

    fn

    ∥∥∥∥∥p

    +

    ∥∥∥∥∥∞∑

    n=m+1

    fn

    ∥∥∥∥∥r

    → 0

    Since∑∞

    n=1 fn ∈ Lp ∩ Lr, this space is a Banach space.Furthermore,Observe that, if

    ‖f − g‖ < �then

    ‖f − g‖q = ‖f − g‖λp ‖f − g‖

    1−λr ≤ ‖f − g‖

    λ ‖f − g‖1−λ < � with 1q

    p+

    1− λr

    Thus, the inclusion map is continuous.�

    4. If 1 ≤ p < r ≤ ∞, Lp + Lr is a Banach space with norm‖f‖ = inf{‖g‖p + ‖h‖r : f = g + h}

    and, if p < q < r, the inclusion map Lq → Lp + Lr is continuous.

    Proof. (April 8th 2018)

    Firstly, note that Lp + Lr is, clearly, a vector space. And the given mapping is a norm due to followingargument.

    Let f ∈ Lp + Lr be given function. Then there exist {gn} ⊂ Lp and {hn} ⊂ Lr such that

    limn→∞

    (‖gn‖p + ‖hn‖r

    )= ‖f‖ gn + hn = f ∀n ∈ N

    ‖f‖ = 0 =⇒ inf{‖g‖p + ‖h‖r : f = g + h} = 0

    =⇒ limn→∞

    (‖gn‖p + ‖hn‖r

    )= 0

    =⇒ limn→∞

    ‖gn‖p = 0 = ‖0‖p ∧ limn→∞ ‖hn‖r = 0 = ‖0‖r=⇒ f = 0 + 0 = 0

    f = 0 =⇒ ‖0‖p + ‖0‖r ≥ inf{‖g‖p + ‖h‖r : f = g + h}=⇒ ‖f‖ = 0

    ‖λf‖ = inf{‖g‖p + ‖h‖r : λf = g + h} = |λ| inf{∥∥∥g

    λ

    ∥∥∥p

    +

    ∥∥∥∥hλ∥∥∥∥r

    : f =g

    λ+h

    λ

    }= |λ| ‖f‖

  • 6

    Let f1, f2 ∈ Lp + Lr. Note that, for given � > 0, there exists g1, g2 and h1, h2 such that fi = gi + hii = 1, 2 and

    ‖g1‖p + ‖h1‖r < ‖f1‖+�

    2‖g2‖p + ‖h2‖r < ‖f2‖+

    2

    Then

    ‖f1 + f2‖ = ‖g1 + g2‖p + ‖h1 + h2‖r ≤ ‖g1‖p + ‖g2‖p + ‖h1‖r + ‖h2‖r < ‖f1‖+ ‖f2‖+ �

    Since � > 0 is arbitrary,

    ‖f1 + f2‖ ≤ ‖f1‖+ ‖f2‖Thus, Lp + Lr is a normed vector space.Let {fn} ⊂ Lp + Lr be a sequence such that

    ∞∑n=1

    ‖fn‖

  • 7

    Proof. (April 10th 2018)

    (1) Let

    a = inf{µ(E) : E ⊂ X ∧ µ(E) > 0}Suppose a > 0 and, letting f ∈ Lp(µ) and An = {x : |f(x)| > n}, observe that

    npµ(An) =

    ∫An

    npdµ <

    ∫An

    |f |pdµ ≤ ‖f‖pp =⇒ µ(An) =‖f‖ppnp

    Note that

    limn→∞

    µ(An) = 0 =⇒ ∃N ∈ N such that µ(An) < a ∀n ≥ N

    =⇒ µ(An) = 0 ∀n ≥ N, then, by the definition of An, ‖f‖∞

  • 8

    Suppose b 1n}), observe that

    1

    nqµ(Bn) <

    ∫Bn

    |f |qdµ ≤∫|f |qdµ = ‖f‖qq

  • 9

    If a = 0, let

    f =∞∑n=1

    2np+1χEn

    Then, by MCT,

    ‖f‖pp =∫|f |pdµ =

    ∞∑n=1

    2npp+1µ(En) <

    ∞∑n=1

    2n(−1p+1) 0 be given and

    E = {x : |f(x)| > ‖f‖∞ − �}

  • 10

    And, ∀q > p, observe that

    ∞ > ‖f‖qq ≥∫E

    |f |qdµ >∫E

    (‖f‖∞ − �)qdµ = µ(E)(‖f‖∞ − �)

    q =⇒ ‖f‖q > µ(E)1q (‖f‖∞ − �)

    =⇒ limq→∞‖f‖q ≥ (‖f‖q − �) limq→∞µ(E)

    1q

    =⇒ limq→∞‖f‖q ≥ ‖f‖q − �

    Since � is arbitrary,

    limq→∞‖f‖q ≥ ‖f‖∞

    On the other hands, observe that

    limq→∞‖f‖q ≤ limq→∞ ‖f‖

    pqp ‖f‖

    1− pq

    ∞ = ‖f‖∞

    Therefore,

    limq→∞‖f‖q = ‖f‖∞

    8. Suppose µ(X) = 1 and f ∈ Lp for some p > 0, so that f ∈ Lq for 0 < q < p.a. log ‖f‖q ≥

    ∫log |f |. (Use exercise 42d in §3.5, with F (t) = et.)

    b. (∫|f |q − 1)/q ≥ log ‖f‖q, and (

    ∫|f |q − 1)/q →

    ∫log |f | as q → 0.

    c. limq→0 ‖f‖q = exp(∫

    log |f |).

    Proof. �

    9. Suppose 1 ≤ p 0 be given. Then for any n ∈ N, letEn,� = {x : |fn(x)− f(x)| ≥ �}

    Then, observe that

    �pµ(En,�) ≤∫En,�

    |fn − f |pdµ ≤ ‖fn − f‖pp → 0 =⇒ µ(En,�)→ 0 as n→∞

    Therefore, fn → f in measure. Hence, by Theorem 2.30, there is a subsequence {fnj} ⊂ {fn} such thatfnj → f a.e.

    On the other hand, suppose that fn → f in measure and |fn| ≤ g ∈ Lp for all n.Note that there is a subsequence {fnk}k such that

    fnk → f a.e.so |f | ≤ g and |fn − f | ≤ 2g ∈ Lp and lim infn→∞ |fn − f | = 0.Observe that, by Fatou’s lemma,∫

    2pgp =

    ∫2pgp +

    ∫|fn − f |p ≤ lim inf

    n→∞

    ∫{2pgp + |fn − f |p} =

    ∫2pgp + lim inf

    n→∞

    ∫|fn − f |p∫

    2pgp =

    ∫2pgp −

    ∫|fn − f |p ≤ lim inf

    n→∞

    ∫{2pgp − |fn − f |p} =

    ∫2pgp − lim sup

    n→∞

    ∫|fn − f |p

  • 11

    Thus,

    lim supn→∞

    ∫|fn − f |p ≤ 0 ≤ lim inf

    n→∞

    ∫|fn − f |p =⇒ ‖fn − f‖p → 0 as n→∞

    10. Suppose 1 ≤ p < ∞. If fn, f ∈ Lp and fn → f a.e., then ‖fn − f‖p → 0 iff ‖fn‖p → ‖f‖p. (UseExercise 20 in §2.3. )

    Proof. (April 11st 2018)

    Suppose ‖fn − f‖p → 0 and, by Minkowski’s Inequality, note that

    | ‖fn‖p − ‖f‖p | ≤ ‖fn − f‖p → 0 =⇒ ‖fn‖p → ‖f‖pConversely, suppose ‖fn‖p → ‖f‖p and note that |fn − f |, |fn|+ |f |, 2|f | ∈ Lp and that

    |fn|+ |f | → 2|f | a.e. limn→∞

    ∫(|fn|+ |f |)p → lim

    n→∞‖|fn|+ |f |‖pp = 2

    p ‖f‖pp

    Therefore, by the Exercise 20 in §2.3, since |fn − f | ≤ |fn|+ |f |,

    limn→∞

    ∫|fn − f |p =

    ∫limn→∞

    |fn − f |p = 0

    Thus,

    ‖fn − f‖p → 0 as n→∞�

    11.

    Proof. �

    12.

    Proof. �

    13. Lp(Rn,m) is separable for 1 ≤ p

  • 12

    And observe that ∫|f(x)− p(x)χRk |pdm ≤ �pµ(Rk) = �p(2k)n

    Thus,

    P ′ = {pχRk : p ∈ P and k ∈ N}

    is dense in Cc(Rn) in p−norm.Since, if PQ is a set of polynomials of n variables with rational coefficients,

    P ′Q = {pχRk : p ∈ PQ and k ∈ N}is countable and for any p ∈ P , there is p′ ∈ PQ such that

    |p′(x)− p(x)| < � ∀x ∈ Rn,observe that ∫

    |p′(x)− p(x)|pχRkdm ≤ �pm(Rk) = �p(2k)n.

    Thus, P ′Q is dense in P′, so is dense in Cc(Rn) and so Lp.

    (2) L∞ is not separable.

    ( =⇒ )For each r ∈ (0,∞), let fr = χBr(0). Then note that

    ‖fr − fs‖∞ = 1 ∀r, s ∈ (0,∞) with r 6= sSuppose that F be a dense subset of L∞, then

    B 12(fr) ∩ F 6= ∅ ∀r ∈ (0,∞)

    Since

    B 12(fr) ∩B 1

    2(fs) = ∅,

    r 7→ B 12(fr)

    is an injective map, so F should be uncountable. �

    14. If g ∈ L∞, the operator T defined by Tf = fg is bounded on Lp for 1 ≤ p ≤ ∞. Its operator norm isat most ‖g‖∞, with equality if µ is semifinite.

    Proof. (April 12th 2018)

    If g = 0,

    ‖Tf‖p = ‖0‖p = 0 = ‖0‖∞ = ‖g‖∞ ∀f ∈ Lp

    When p

  • 13

    |fg| = |f ||g| ≤ ‖f‖∞ ‖g‖∞ =⇒ ‖fg‖∞ ≤ ‖f‖∞ ‖g‖∞thus,

    ‖T‖ ≤ ‖g‖∞Conversely, when 1 ≤ p ≤ ∞, if µ is semifinite, then ∀� > 0 there is a set A ⊂ X, such that

    0 < µ(A) ‖g‖∞ − �Then, note that χA ∈ Lp and

    ‖TχA‖p =(∫|gχA|pdµ

    ) 1p

    >

    ((‖g‖∞ − �)

    p

    ∫|χA|pdµ

    ) 1p

    = (‖g‖∞ − �) ‖χA‖p

    or

    ‖TχA‖∞ = ‖gχA‖∞ > (‖g‖∞ − �) ‖χA‖∞Thus,

    ‖T‖ ≥‖TχA‖p‖χA‖p

    > ‖g‖∞ − �

    Since � > 0 is arbitrary,

    ‖T‖ ≥ ‖g‖∞Therefore,

    ‖T‖ = ‖g‖∞�

    15.

    Proof. �

    16.

    Proof. �

    17. With the notation as in Theorem 6.14, if µ is semifinite, q 0 and hence Sg is σ−finite.

    Proof. .

    If not, ∃δ > 0 such thatµ(E�) =∞ ∀0 < � ≤ δ

    where

    E� = {x : |g(x)| > �}.Let

    λ = sup{α > 0 : µ(Eα) =∞}.

    (λ exists since µ is semifinite, so {α > 0 : µ(Eα) =∞} is bounded.)

  • 14

    Note that, when f = 1µ(E

    λ+ 1n)1pχE

    λ+ 1n

    sgn(g),

    ‖f‖p = 1So

    Mq(g) ≥∣∣∣∣∫ fg∣∣∣∣ = 1

    µ(Eλ+ 1n)1p

    ∫Eλ+ 1n

    |g|dµ > µ(Eλ+ 1n)1−

    1p

    (λ+

    1

    n

    )then

    ∞ > Mq(g) ≥ limn→∞

    µ(Eλ+ 1n)1−

    1p

    (λ+

    1

    n

    )= λµ(Eλ)

    1− 1p =∞.

    And it is a contradiction.�

    18.

    Proof. �

    19.

    Proof. �

    20. Suppose supn ‖fn‖p 0 such that

    ∫E|g|p < � whenever µ(E) < δ, (ii)A ⊂ X such that µ(A) 0 be given and let q be the conjugate exponent of p. And let A = supn ‖fn‖p + 1.

    Note that

    ν(E) =

    ∫E

    |g|q

    is a finite measure and is absolutely continuous with respect to µ. Thus, there is δ > 0 such that

    ∀E with µ(E) < δ =⇒∫E

    |g|q =∣∣∣∣∫E

    |g|q∣∣∣∣ < 12 ( �4A)q

    Now, let

    Bn = {x : |g(x)|q ≥1

    n} ∀n

    then ⋃n

    Bn = {x : |g(x)|q > 0} =let= B.

  • 15

    therefore,

    limn→∞

    ∫Bcn

    |g|q = limn→∞

    (∫|g|q −

    ∫Bn

    |g|q)

    =

    ∫|g|q − lim

    n→∞

    ∫Bn

    |g|q =∫|g|q −

    ∫B

    |g|q = 0

    Thus, there exists N ∈ N such that ∫BN

    |g|q < 12

    ( �4A

    )qNote that, since

    µ(BN)

    N=

    ∫BN

    1

    N≤∫BN

    |g|q ≤∫|g|q

  • 16

    which means

    |g(fn − f)| ≤ 2|g|M a.e. ∀nThen, by DCT,

    limn→∞

    ∫|g(fn − f)| = 0

    Observe that

    0 ≤∣∣∣∣∫ gfn − ∫ gf ∣∣∣∣ ≤ ∫ |g(fn − f)|

    Therefore,

    limn→∞

    ∣∣∣∣∫ gfn − ∫ gf ∣∣∣∣ = 0.So, when p =∞, if µ is σ−finite, (a) is true.

    21. If 1 < p

  • 17

    Proof. .

    a. Recall that L2 space is a Hilbert space with respect to the inner product

    〈f, g〉 =∫[0,1]

    fgdm.

    And also note that

    〈cos 2πnx, cos 2πmx〉 =∫[0,1]

    cos 2πnx cos 2πmxdm = 0 if n 6= m.

    Therefore { cos 2πnx‖cos 2πnx‖}n is an orthonormal sequence.Then, by Exercise 5.63,

    cos 2πnx

    ‖cos 2πnx‖→ 0 weakly in L2.

    Assume that cos 2πnx→ 0 in measure. Then there exists a subsequence (cos 2πnjx)2 → 0 a.e. Observethat, since

    |cos 2πnjx|2 ≤ 1 ∀j on [0, 1]and 1 ∈ L2([0, 1]), by DCT,

    limj→∞

    ∫[0,1]

    (cos 2πnjx)2dm =

    ∫[0,1]

    0dm = 0.

    However,

    limj→∞

    ∫[0,1]

    (cos 2πnjx)2dm = lim

    n→∞

    1

    8

    (sin 4πnjπnj

    + 4

    )=

    1

    2

    Thus, it is a contradiction.

    b.Let � > 0 and x ∈ (0, 1] be given and note that ∃N ∈ N such that 1

    N< x, then

    |fn(x)| = |nχ(0, 1n)(x)| = 0 < � ∀n ≥ N.

    Since {0} has measure 0, fn → 0 a.e.Now, let En(�) = {x ∈ [0, 1] : |fn(x)| > �}. Note that

    limn→∞

    µ({x ∈ [0, 1] : |fn(x)| > �}) = limn→∞

    µ({x ∈ [0, 1] : |nχ(0, 1n)(x)| > �}) = 0.

    Thus, fn → 0 in measure.

    However, for any p, note that 1 ∈ Lq([0, 1]) (q is a conjugate exponent of p) and

    limn→∞

    ∫fndm = lim

    n→∞

    ∫nχ(0, 1

    n)dm = 1.

    Therefore, fn 6→ 0 weakly in Lp. �

    23.

    Proof. �

    24.

  • 18

    Proof. �

    25.

    Proof. �

    26.

    Proof. �

    27.

    Proof. �

    28.

    Proof. �

    29.

    Proof. �

    30.

    Proof. �

    31. (A Generalized Höder Inequality) Suppose that 1 ≤ pj ≤ ∞ and∑n

    1 p−1j = r

    −1 ≤ 1. If fj ∈ Lpjfor j = 1, . . . , n, then

    ∏n1 fj ∈ Lr and ‖

    ∏n1 fj‖r ≤

    ∏n1 ‖fj‖pj . (First do the case n = 2.)

    Proof. .

    When n = 1, it is obvious. When n = 2, then with 1 < p1, p2

  • 19

    Now assume that ∥∥∥∥∥n∏1

    fj

    ∥∥∥∥∥r

    ≤n∏1

    ‖fj‖pj withn∑j=1

    1

    pj=

    1

    r′≤ 1.

    And let fn+1 ∈ Lpn+1 and∑n+1

    11pj

    = 1r≤ 1, so 1

    r′+ 1

    pn+1= 1

    r≤ 1. Observe that, by the case when n = 2,∥∥∥∥∥

    n+1∏1

    fj

    ∥∥∥∥∥r

    ∥∥∥∥∥n∏1

    fj

    ∥∥∥∥∥r

    ‖fn+1‖pn+1 ≤n+1∏1

    ‖fj‖pj

    Therefore, ∥∥∥∥∥n∏1

    fj

    ∥∥∥∥∥r

    ≤n∏1

    ‖fj‖pj ∀n ∈ N

    32. Suppose that (X,M, µ) and (Y,N , ν) are σ−finite measure spaces and K ∈ L2(µ× ν). If f ∈ L2(ν),the integral Tf(x) =

    ∫K(x, y)f(y)dν(y) converge absolutely for a.e. x ∈ X; moreover, Tf ∈ L2(µ) and

    ‖Tf‖2 ≤ ‖K‖2 ‖f‖2.

    Proof. .

    Observe that K ∈ L2(µ× ν), so by Toneli,∫|K(x, y)|2d(µ× ν) =

    ∫ (∫|K(x, y)|2dν(y)

    )dµ(x)

  • 20

    33. (Exercise 8.5) If s : Rn × Rn → Rn is defined by s(x, y) = x− y, then s−1(E) is Lebesgue measurablewhenever E is Lebesgue measurable. (For n = 1, draw a picture of s−1(E) ⊂ R2. It should be clear thatafter rotation through an angle π

    4, s−1(E) becomes F × R where F = {x :

    √2x ∈ E}, and Theorem 2.44

    can be applied. The same idea works in higher dimensions.)

    Proof. .

    Observe that, when n = 1,

    s−1((a, b)) = {(x, y) ∈ R2 : y = x− c ∀c ∈ (a, b)}.Rotating the set counterclockwise centered at (0, 0) through π

    4, you get

    (

    √2a

    2,

    √2b

    2)× R

    With same idea, if E ⊂ R is Lebesgue measurable set, after rotating s−1(E) counterclockwise centeredat the origin, through π

    4, we have F × R where F = {x :

    √2x ∈ E}. Since F × R is Lebesgue measurable

    and measurability is independent of scaling and rotation, s−1(E) is also Lebesgue measurable.

    In general, when n is arbitrary natural number, let E ⊂ Rn be a Lebesgue measurable set. Then, withappropriate rotational operator, we rotate s−1(E) and get

    (Fn × Rn) =let {x ∈ Rn :√

    2nx ∈ E} × Rn

    Since Lebesgue measuability is independent of scaling and rotating, Fn is Lebesgue measurable, and sos−1(E) is a Lebesgue measurable set. �

    34. (Exercise 8.7) If f is locally integrable on Rn and g ∈ Ck has compact support, then f ∗ g ∈ Ck.

    Proof. .

    Recall

    f ∗ g =∫f(y)g(x− y)dy.

    Let Sg = {x : g(x) 6= 0} = supp(g) and let S = {x− y : y ∈ Sg}.Now, observe that, for all 0 ≤ j ≤ k,

    ∂j

    ∂xj(f ∗ g) =

    ∫f(y)

    ∂jg(x− y)∂xj

    dy =

    ∫S

    f(y)∂jg(x− y)

    ∂xjdy

    Since S is reflection and translation of a Lebesgue measurable set Sg, S is also Lebesgue measurable, sof is integrable over S. In addition, since g ∈ Ck, ∂jg is continuous, so is integrable over compact set S.Therefore, fτx(

    ∂jg∂xj

    ) is integrable over S. Therefore, ∂j(f∗g)∂xj

    is continuous for all 0 ≤ j ≤ k, so

    f ∗ g ∈ Ck

    35.

    Proof. �

    36.

  • 21

    Proof. �

    37.

    Proof. �

    38.

    Proof. �

    39.

    Proof. �

    40.

    Proof. �

    41.

    Proof. �

    42.

    Proof. �

    43.

    Proof. �

    44.

    Proof. �

    45.

    Proof. �

    46.

    Proof. �

    47.

    Proof. �

    48.

  • 22

    Proof. �

    49.

    Proof. �

    50.

    Proof. �

    51.

    Proof. �

    52.

    Proof. �

    53.

    Proof. �

    54.

    Proof. �

    55.

    Proof. �

    56.

    Proof. �

    57.

    Proof. �

    58.

    Proof. �

    59.

    Proof. �

    60.

  • 23

    Proof. �

    61.

    Proof. �

    62.

    Proof. �

    63.

    Proof. �

    64.

    Proof. �

    65.

    Proof. �

    66.

    Proof. �

    67.

    Proof. �

    68.

    Proof. �

    69.

    Proof. �

    70.

    Proof. �

    71.

    Proof. �

    72.

  • 24

    Proof. �

    73.

    Proof. �

    74.

    Proof. �